Question and Answers Forum

All Questions      Topic List

Others Questions

Previous in All Question      Next in All Question      

Previous in Others      Next in Others      

Question Number 80504 by Rio Michael last updated on 03/Feb/20

Solve the system of congruences  x ≡ 2 (mod 3)  x ≡ 5( mod 7)

$$\mathrm{Solve}\:\mathrm{the}\:\mathrm{system}\:\mathrm{of}\:\mathrm{congruences} \\ $$$${x}\:\equiv\:\mathrm{2}\:\left(\mathrm{mod}\:\mathrm{3}\right) \\ $$$${x}\:\equiv\:\mathrm{5}\left(\:\mathrm{mod}\:\mathrm{7}\right) \\ $$$$\: \\ $$

Commented by mr W last updated on 03/Feb/20

x=21n+5

$${x}=\mathrm{21}{n}+\mathrm{5} \\ $$

Commented by Rio Michael last updated on 03/Feb/20

please check this sir:   x ≡ 2(mod 3) ⇒ x = 3k + 2   3k + 2 ≡ 5(mod 7) ⇒ 3k ≡ 3(mod 7) k has no   solution here sir!  for chinese remainder theorem   R_1  = 7 and R_2  = 3   7X_1 ≡ 1 (mod 3) ⇒ X_1  ≡ 1 (mod 3)  3X_2  ≡ 1 (mod 7) ⇒ X_2  ≡ 5(mod 7)  x ≡ R_1 a_1 X_1  + R_2 a_2 X_2 (mod n_1 n_2 )  x ≡ (7)(2)(1) + (3)(5)(5) (mod 21)  x ≡ 5(mod 21)

$${please}\:{check}\:{this}\:{sir}: \\ $$$$\:{x}\:\equiv\:\mathrm{2}\left({mod}\:\mathrm{3}\right)\:\Rightarrow\:{x}\:=\:\mathrm{3}{k}\:+\:\mathrm{2}\: \\ $$$$\mathrm{3}{k}\:+\:\mathrm{2}\:\equiv\:\mathrm{5}\left({mod}\:\mathrm{7}\right)\:\Rightarrow\:\mathrm{3}{k}\:\equiv\:\mathrm{3}\left({mod}\:\mathrm{7}\right)\:{k}\:{has}\:{no}\: \\ $$$${solution}\:{here}\:{sir}! \\ $$$${for}\:{chinese}\:{remainder}\:{theorem} \\ $$$$\:{R}_{\mathrm{1}} \:=\:\mathrm{7}\:{and}\:{R}_{\mathrm{2}} \:=\:\mathrm{3} \\ $$$$\:\mathrm{7}{X}_{\mathrm{1}} \equiv\:\mathrm{1}\:\left({mod}\:\mathrm{3}\right)\:\Rightarrow\:{X}_{\mathrm{1}} \:\equiv\:\mathrm{1}\:\left({mod}\:\mathrm{3}\right) \\ $$$$\mathrm{3}{X}_{\mathrm{2}} \:\equiv\:\mathrm{1}\:\left({mod}\:\mathrm{7}\right)\:\Rightarrow\:{X}_{\mathrm{2}} \:\equiv\:\mathrm{5}\left({mod}\:\mathrm{7}\right) \\ $$$${x}\:\equiv\:{R}_{\mathrm{1}} {a}_{\mathrm{1}} {X}_{\mathrm{1}} \:+\:{R}_{\mathrm{2}} {a}_{\mathrm{2}} {X}_{\mathrm{2}} \left({mod}\:{n}_{\mathrm{1}} {n}_{\mathrm{2}} \right) \\ $$$${x}\:\equiv\:\left(\mathrm{7}\right)\left(\mathrm{2}\right)\left(\mathrm{1}\right)\:+\:\left(\mathrm{3}\right)\left(\mathrm{5}\right)\left(\mathrm{5}\right)\:\left({mod}\:\mathrm{21}\right) \\ $$$${x}\:\equiv\:\mathrm{5}\left({mod}\:\mathrm{21}\right) \\ $$$$ \\ $$

Terms of Service

Privacy Policy

Contact: info@tinkutara.com